LSAT and Law School Admissions Forum

Get expert LSAT preparation and law school admissions advice from PowerScore Test Preparation.

User avatar
 Dave Killoran
PowerScore Staff
  • PowerScore Staff
  • Posts: 5852
  • Joined: Mar 25, 2011
|
#80541
This game is also discussed in our Podcast: LSAT Podcast Episode 70: The May 2020 LSAT-Flex Logic Games Section

Complete Question Explanation
(The complete setup for this game can be found here: lsat/viewtopic.php?t=33051)

The correct answer choice is (D).

This is a List question, but it is an annoying one since it's a partial List question. Additionally, since it's a Cannot Be True questions, four of the answers will work; only one—the correct answer—will not work. This should make you suspect that the answer will not be revealed by a simple application of the rules (since we can't see all the cities in each semester). You'll have to go farther and look at how they combine to solve this question.

There are only three rules in the game, so that helps in this question. The first two rules give you solid "if-then" scenarios that allow you to place cities and see consequences, which is beneficial. The third rule is less helpful since M and V could be part of the "unlisted" cities in the answers. So, focus on the first two rules at the start since they allow you see consequences, and just as a speed consideration, perhaps look at those answers that meet the conditions of at least one of the first two rules first, then look at answers that don't thereafter. This would create an order of answer consideration that goes: (A)-(D)-(E)-(B)-(C). With that in mind, let's look at each answer individually.


Answer choice (A): This answer places H in the fall, which triggers the first rule and forces M to host in the fall as well. Since V is in the spring, that then satisfies the third rule. and, V in the spring also triggers the second rule, forcing T into hosting in the spring as well. O is still unplaced, but there is room in the fall, leaving us with the following solution:

  • Fall: H, M, O
    Spring: H, V, T
Since this solution does not violate any rules, it could occur and thus this answer choice is incorrect.

Answer choice (B): Enjoyably, if you produced the hypothetical above for answer choice (A), you can use that to prove this answer choice is possible:

  • Fall: H, M, O
    Spring: H, V, T
If you didn't see that connection, then note that this answer does not meet the conditions of either of the first rules, so it's only a matter of satisfying the third rule and then making sure you haven't missed anything else.

Answer choice (C): There's no immediate trigger from either of the first two rules in play, and there is enough "room" in the spring for the second rule to execute so at first glance this answer looks likely to be possible to occur. And it is:

  • Fall: O, M, H
    Spring: M, T, V

Answer choice (D): This is the correct answer choice. With H hosting in the fall, then from the first rule M must also host in the fall. That completes the fall roster at H, T, and M. Thus, both O and V must host in the spring, along with M, which completes the spring roster as well. However, this causes an issue with the second rule since when V hosts in the spring then T must host in the spring. But, there are no open meetings in the spring for T. Consequently, there are not enough meetings in the spring to accommodate all the required cities, and this answer cannot occur.

Answer choice (E): This answer triggers each of the first two rules, but also satisfies the third rule based on the partial list provided. Since there is enough space in each term to accommodate the consequences of the first two rules, this answer choice produces a workable solution and is thus incorrect:

  • Fall: H, V, M
    Spring: V, T, O

Get the most out of your LSAT Prep Plus subscription.

Analyze and track your performance with our Testing and Analytics Package.